Jump to content

Dieu Ha's Content

There have been 29 items by Dieu Ha (Search limited from 05-06-2020)



Sort by                Order  

#346272 Giải hệ $\left\{\begin{array}{l}...

Posted by Dieu Ha on 12-08-2012 - 20:45 in Phương trình, hệ phương trình và bất phương trình

Giải hệ phương trình $\left\{\begin{array}{l}x^{2}+y^{2}-2x-y=2 \\2x^{2}+xy-y^{2}=1 \end{array}\right.$



#344947 Giải hệ $\left\{\begin{array}{l}...

Posted by Dieu Ha on 08-08-2012 - 23:22 in Phương trình, hệ phương trình và bất phương trình

Giải hệ $\left\{\begin{array}{l}x^{4}+y^{4}=1 \\x^{5}+y^{5}=1 \end{array}\right.$



#315915 Tam giác ABC cân tại A nội tiếp (O). M là trung điểm AC. BM cắt (O) tại Q. C/...

Posted by Dieu Ha on 11-05-2012 - 22:50 in Hình học

1. Tam giác ABC cân tại A nội tiếp (O). M là trung điểm AC. BM cắt (O) tại Q. C/m BQ $\geq$ 2AQ
2. Tam giác ABC vuông tại A. Trên tia AB, AC lấy E, F tương ứng sao cho BE = CF = BC. Chứng minh rằng với mọi M nằm trên đường tròn đường kính BC ta đều có MA + MB + MC $\leq$ EF
3. Tam giác ABC nội tiếp (O;R). Tìm M thuộc cung BC không chứa A sao cho 2011.MB +2012.MC đạt giá trị nhỏ nhất



#307328 Chứng minh SK là tiếp tuyến của (O)

Posted by Dieu Ha on 31-03-2012 - 19:44 in Hình học

Cho đường tròn (O). Hai đường kính AB và MK không vuông góc với nhau. Trên đường kính AB lấy E,F sao cho OE = OF. Vẽ dây MC đi qua E, dây MD đi qua F. Đường thẳng CD cắt đường thẳng AB tại S.
Chứng minh SK là tiếp tuyến của (O)



#302079 $\left\{\begin{array}{l} x^{2}-xy+y^{2}=3(x-y)\...

Posted by Dieu Ha on 03-03-2012 - 21:39 in Phương trình, hệ phương trình và bất phương trình

Bài 1:
$\left\{\begin{array}{l}x-\frac{1}{x^{3}} = y-\frac{1}{y^{3}}\\ (x - 4y)(y-2x-4) = 36\end{array}\right.$
Bài 2:
$\left\{\begin{array}{l} \sqrt{\frac{2x}{y}}+\sqrt{\frac{2y}{x}} = 3\\ x-y+xy = 3 \end{array}\right.$
Bài 3:
$\left\{\begin{array}{l} (4x^{2}+1)x + (y-3)\sqrt{5-2y}=0 \\ 3x^{2}+4y^{2} = 4\end{array}\right.$
Bài 4:
$\left\{\begin{array}{l} x^{2}=(y+1)(x+1)^{2}\\y = \sqrt{\frac{2x^{2}+1}{x+1}} \end{array}\right.$
Bài 5:
$\left\{\begin{array}{l} x^{2}-xy+y^{2}=3(x-y)\\x^{2}+xy +y^{2}=7(x-y)^{2}\end{array}\right.$
Bài 6:
$\left\{\begin{array}{l}x^{2}y+2x-3y+1=0 \\ 2x^{2}y+y^{2}(x-4) + 2x +y = 0\end{array}\right.$



#297372 Chứng minh không thể dùng hai hình chữ I (2 ô vuông), 3 hình chữ T (4 ô vuông...

Posted by Dieu Ha on 29-01-2012 - 22:32 in Hình học

Chứng minh không thể dùng hai hình chữ I (2 ô vuông), 3 hình chữ T (4 ô vuông) để xếp kín 1 hình vuông 4x4



#297370 Chứng minh $(x+y+z)^{2}(xy+yz+xz)^{2}\leq 3(x^{2}+xy+y^{2})(y^{2}+y...

Posted by Dieu Ha on 29-01-2012 - 22:22 in Bất đẳng thức và cực trị

B1: Chứng minh:
$(x+y+z)^{2}(xy+yz+xz)^{2}\leq 3(x^{2}+xy+y^{2})(y^{2}+yz+z^{2})(x^{2}+xz+z^{2})$
với mọi $x,y,z\geq 0$
B2: $a,b,c\geq 0$ Chứng minh:
$\frac{(a+b)(b+c)(c+a)}{abc}\geq \frac{8(a+b+c)}{\sqrt{ab}+\sqrt{bc}+\sqrt{ca}}$
B3: $a,b,c>0$ Chứng minh:
$\sqrt{\frac{a}{b+a}}+\sqrt{\frac{b}{b+c}}+\sqrt{\frac{c}{c+a}}\leq \frac{3}{\sqrt{2}}$
B4: x,y>0 x+y+xy = 3
$\frac{3x}{1+y}+\frac{3y}{1+x}+\frac{xy}{x+y}\leq x^{2}+y^{2}+\frac{3}{2}$
B5: x+y+z = 3 CM:
$x^{2}y+y^{2}z+z^{2}x\leq 4$



#279042 Hình Học 9

Posted by Dieu Ha on 15-10-2011 - 13:41 in Hình học

1. Cho 2 đường tròn (O) và (O') cắt nhau tại M, N. PQ là tiếp tuyến chung của 2 đường tròn này (P thuộc (O) và Q thuộc (O') ). MN cắt PQ tại I (N nằm giữa I,M). PN kéo dài cắt (O') tại K. CM MQ là phân giác của góc PMK
2. Cho (O) và dây AB. I là điểm chính giữa cung nhỏ AB. 2 dây IC, ID của (O) cắt AB lần lượt tại M,N
a) Chứng minh IA là tiếp tuyến của đường tròn ngoại tiếp tam giác AMC
b) Chứng minh đường tròn ngoại tiếp tam giác AMC và đtròn ngt tgiác AND tiếp xúc với nhau tại A



#278740 Một bài toán diện tích

Posted by Dieu Ha on 12-10-2011 - 22:08 in Hình học

Đề bài: Tính diện tích một ngũ giác lồi, biết diện tích của của các tam giác có ba đỉnh là các đỉnh liên tiếp của ngũ giác đều bằng 1



#278737 Giải hệ phương trình

Posted by Dieu Ha on 12-10-2011 - 22:06 in Đại số

\[\left\{ {
\begin{array}{l}
x^4 + 2{\rm{x}}^3 y + x^2 y^2 = 2{\rm{x}} + 9 \\
x^2 + 2{\rm{x}}y = 6{\rm{x}} + 6 \\
\end{array}} \right.\]



#274870 Một số bài toán về đường tròn nội tiếp tam giác!

Posted by Dieu Ha on 02-09-2011 - 09:08 in Hình học

Mọi người giúp mình các bài này với:
1. Tam giác ABC vuông tại A, AH là đường cao. Gọi $(O,r);(O_1 ,r_1 );(O_2 ,r_2 )$ lần lượt nội tiếp tam giác ABC, tam giác ABH, tam giác ACH
a) CMR ${\rm{r}} + r_1 + r_2 = AH$
b)CMR $r^2 = r_1 ^2 + r_2 ^2$
c)Tính $OO_2$ biết AB = 3, AC = 4
2. Đường tròn nội tiếp tam giác ABC tiếp xúc với AB tại D sao cho AC.BC=2AD.DB
Chứng minh tam giác ABC vuông tại C
3.Gọi h là đường cao ứng với cạnh huyền, r là bán kính đường tròn nội tiếp tam giác vuông. CM $2 < \dfrac{h}{r} \le \sqrt 2$



#274869 Một vài bài toán giải phương trình!

Posted by Dieu Ha on 02-09-2011 - 08:27 in Phương trình, hệ phương trình và bất phương trình

Đã ai làm được câu g chưa ah?
Đề có chút sai sót, mng thông cảm
Đề hoàn chỉnh đây ạ:
g) $\sqrt {x - 1} + x - 3 = \sqrt {4{\rm{x}}^2 - 10{\rm{x}} + 16} $



#274648 Một vài bài toán giải phương trình!

Posted by Dieu Ha on 31-08-2011 - 20:42 in Phương trình, hệ phương trình và bất phương trình

Cảm ơn các bạn nhé!



#274639 Một vài bài toán giải phương trình!

Posted by Dieu Ha on 31-08-2011 - 19:58 in Phương trình, hệ phương trình và bất phương trình

a) $2(x^2 - 3{\rm{x}} + 2) = 3\sqrt {x^3 + 8} $
b) $4{\rm{x}}^2 - 8{\rm{x}} + \sqrt {2{\rm{x}} + 3} = 1$
c) $2{\rm{x}}^2 - 6{\rm{x}} - 1 = \sqrt {4{\rm{x}} + 5} $
d) $\sqrt {8{\rm{x}} + 1} + \sqrt {3{\rm{x}} - 5} = \sqrt {7{\rm{x}} + 4} + \sqrt {2{\rm{x - 2}}} $
e) $x^2 + \sqrt {x + 5} = 5$
f) $3 + \sqrt {3 + \sqrt x } = x$
g) $\sqrt {x - 1} + x - 3 = \sqrt {4{\rm{x}}^2 - 10{\rm{x}} + 16} $
Mng giúp mình mấy bài giải phtr này với



#274294 Giải phương trình!

Posted by Dieu Ha on 28-08-2011 - 17:50 in Phương trình, hệ phương trình và bất phương trình

Giải phương trình:
$5\sqrt {2{\rm{x}}^3 + 16} = 2(x^2 + 8)$



#274076 Chứng minh một bài bất đẳng thức

Posted by Dieu Ha on 26-08-2011 - 21:53 in Bất đẳng thức và cực trị

Cho m,n là các số nguyên dương và $\sqrt 7 - \dfrac{m}{n} > 0$
Chứng minh: $\sqrt 7 - \dfrac{m}{n} > \dfrac{1}{{mn}}$



#273456 Chứng minh 1 bài bất đẳng thức lớp 9!

Posted by Dieu Ha on 21-08-2011 - 21:47 in Bất đẳng thức và cực trị

Cho x+y+z=2
Chứng minh $\sqrt {x^2 + \dfrac{1}{{x^2 }}} + \sqrt {y^2 + \dfrac{1}{{y^2 }}} + \sqrt {z^2 + \dfrac{1}{{z^2 }}} \ge \dfrac{{\sqrt {97} }}{2}$



#271797 Giải bài phương trình vô tỷ!

Posted by Dieu Ha on 10-08-2011 - 23:04 in Phương trình, hệ phương trình và bất phương trình

Có bài phương trình vô tỷ này em nghĩ mãi ko ra, mọi người giúp em giải với nhé
Phương trình đây:
$\sqrt {x + 3} + \sqrt {2{\rm{x}} + 1} = \sqrt {2{\rm{x}}^2 + 7{\rm{x}} + 3}$



#270711 CM 2 góc bằng nhau

Posted by Dieu Ha on 03-08-2011 - 21:23 in Hình học

Posted Image
Vẽ $AK \bot AB(K \in BC)$
$ = > \widehat{K{\rm{A}}H} = 90^0 $
$= > \widehat{K{\rm{A}}C} + \widehat{CAH} = 90^0 $
Mà $\widehat{ACB} - \widehat{B} = 90^0 = > \widehat{K{\rm{A}}C} + \widehat{K{\rm{A}}B} - \widehat{B} = 90^0 = > \widehat{K{\rm{A}}C} + 90^0 - \widehat{B} = 90^0 = > \widehat{K{\rm{A}}C} = \widehat{B}$
Mặt khác $\widehat{K{\rm{A}}C} + \widehat{CAH} = \widehat{K{\rm{AH}}}{\rm{ = 90}}^0 = > \widehat{B} + \widehat{CAH} = 90^0$
Mà $\widehat{B} + \widehat{BCH} = 90^0 = > \widehat{CAH} = \widehat{BCH}$ (đpcm)

Ps: Hình hơi nhỏ, bạn thông cảm ;))



#270588 Giải phương trình khó!

Posted by Dieu Ha on 02-08-2011 - 19:09 in Phương trình, hệ phương trình và bất phương trình

$x \ge 1$
Trường hợp $x=2$ là lời giải.
Nếu $1 \le x < 2$ thì $\sqrt[3]{{x+6}}+\sqrt{x-1}> x^{2}-1$
Nếu $x>2$ thì $\sqrt[3]{{x+6}}+\sqrt{x-1}< x^{2}-1$


Giải thích rõ hơn các bất đẳng thức xảy ra trong bài đc ko?, em chưa rõ.
Cảm ơn vì đã giúp đỡ! ;))



#270587 Một bài toán khó!

Posted by Dieu Ha on 02-08-2011 - 19:02 in Đại số

Đề bài : Cho a,b,c thỏa mãn:
$\dfrac{b^2 + c^2 - a^2 }{2bc} + \dfrac{c^2 + a^2 - b^2 }{2ac} + \dfrac{a^2 + b^2 - c^2 }{2ab} = 1$
Giải :
$\dfrac{b^2 + c^2 - a^2 }{2bc} + \dfrac{c^2 + a^2 - b^2 }{2ac} + \dfrac{a^2 + b^2 - c^2 }{2ab} = 1$

$ \Leftrightarrow (\dfrac{c^2 + a^2 - b^2 }{2ac} - 1) + (\dfrac{a^2 + b^2 - c^2 }{2ab} - 1) + (\dfrac{b^2 + c^2 - a^2 }{2bc} + 1) = 0$

$ \Leftrightarrow \dfrac{c^2 + a^2 - b^2 - 2ac }{2ac} + \dfrac{a^2 + b^2 - c^2 - 2ab}{2ab} + \dfrac{b^2 + c^2 - a^2 + 2bc}{2bc} = 0$

$\Leftrightarrow \dfrac{( a - c )^2 - b^2}{2ac} + \dfrac{( a - b )^2 - c^2}{2ab} + \dfrac{( b + c )^2 - a^2}{2bc} = 0$

$\Rightarrow \dfrac{(a - c - b)(a - c + b)}{2ac}+\dfrac{( a - b - c )(a - b + c)}{2ab} + \dfrac{( b + c - a )( b + c + a )}{2bc } = 0$

$\Leftrightarrow \dfrac{a - b - c}{2}[\dfrac{a - c + b }{ac} + \dfrac{a - b + c}{ab} - \dfrac{a + b + c}{bc}] = 0$

$\Rightarrow (a - b - c)(\dfrac{ab - bc + b^2}{abc} + \dfrac{ac - bc + c^2 }{abc} - \dfrac{a^2 + ab + ac}{abc}) = 0$

$\Leftrightarrow \dfrac{( a - b - c )(ab - bc + b^2 + ac - bc + c^2 - a^2 - ab - ac)}{abc} = 0$

$\Leftrightarrow \dfrac{(a - b - c)(b^2 - 2bc + c^2 - a^2 )}{abc} = 0$

$ \Rightarrow (a - b - c)[( b - c)^2 - a^2] = 0 \Leftrightarrow (a - b - c )( b - c -a )( b - c + a ) = 0$

$ \Rightarrow \left[\begin{array}{l} a = b + c\\a = b - c\\ a = c - b\end{array}\right.$
Với mỗi giá trị a như vậy, thay vào từng biểu thức trên. Ta sẽ có điều phải chứng minh.

P/S: Ai có cách ngắn hơn thì đóng góp nhé !


Thks bạn nhiều nhé! ;))



#270566 Một bài toán đại số lớp 9 khó!

Posted by Dieu Ha on 02-08-2011 - 16:06 in Đại số

Cho $x_1 ,x_2 ,...,x_{11}$ thỏa mãn $1 < x_1 < x_2 < ... < x_{11} < 1000$
Chứng minh rằng $\exists i \in \left\{ {1,2,3,...,10} \right\}$ sao cho:
$x_{i + 1} - x_1 - 1 < 3\sqrt[3]{{x{}_ix_1 + 1}}$

Mod: bạn coi lại đề với.



#270562 Giải phương trình khó!

Posted by Dieu Ha on 02-08-2011 - 15:53 in Phương trình, hệ phương trình và bất phương trình

Giúp em bài giải pt này với:

$\sqrt[3]{{x + 6}} + \sqrt {x - 1} = x^2 - 1$



#270526 Một bài toán khó!

Posted by Dieu Ha on 02-08-2011 - 08:49 in Đại số

Cho a,b,c thỏa mãn:
$\dfrac{{b^2 + c^2 - a^2 }}{{2bc}} + \dfrac{{c^2 + a^2 - b^2 }}{{2ac}} + \dfrac{{a^2 + b^2 - c^2 }}{{2ab}} = 1$
Chứng minh thì hai phân thức có giá trị bằng 1 và 1 phân thức có giá trị là -1

Giúp em nhé mng!



#270524 Một bài toán bất đẳng thức!

Posted by Dieu Ha on 02-08-2011 - 08:40 in Bất đẳng thức và cực trị

Chứng minh rằng:
$A = \dfrac{1}{{\left( {1 + \sqrt 3 } \right)^3 }} + \dfrac{1}{{\left( {\sqrt 3 + \sqrt 5 } \right)^3 }} + ... + \dfrac{1}{{\left( {\sqrt {2003} + \sqrt {2005} } \right)^3 }} < \dfrac{{246}}{{2007}}$
Các bác giúp em cái, thks nhiều :D